2023 AMC 12A Problems/Problem 15

Revision as of 01:48, 10 November 2023 by Cantalon (talk | contribs) (Solution 1)

Question

Usain is walking for exercise by zigzagging across a $100$-meter by $30$-meter rectangular field, beginning at point $A$ and ending on the segment $\overline{BC}$. He wants to increase the distance walked by zigzagging as shown in the figure below $(APQRS)$. What angle $\theta$$\angle PAB=\angle QPC=\angle RQB=\cdots$ will produce in a length that is $120$ meters? (This figure is not drawn to scale. Do not assume that he zigzag path has exactly four segments as shown; there could be more or fewer.)

[someone add diagram]

$\textbf{(A)}~\arccos\frac{5}{6}\qquad\textbf{(B)}~\arccos\frac{4}{5}\qquad\textbf{(C)}~\arccos\frac{3}{10}\qquad\textbf{(D)}~\arcsin\frac{4}{5}\qquad\textbf{(E)}~\arcsin\frac{5}{6}$

Solution 1

By "unfolding" line $APQRS$ into a straight line, we get a right angled triangle $ABS$.

$cos(\theta)=\frac{120}{100}$

$\theta=\boxed{\textbf{(A) } \cos^{-1}\left(\frac{5}{6})\right}$ (Error compiling LaTeX. Unknown error_msg)

~lptoggled

Video Solution 1 by OmegaLearn

https://youtu.be/NhUI-BNCIUE


See also

2023 AMC 12A (ProblemsAnswer KeyResources)
Preceded by
Problem 14
Followed by
Problem 16
1 2 3 4 5 6 7 8 9 10 11 12 13 14 15 16 17 18 19 20 21 22 23 24 25
All AMC 12 Problems and Solutions

The problems on this page are copyrighted by the Mathematical Association of America's American Mathematics Competitions. AMC logo.png